Homesweet Learning helps students learn!
AMC12 2021a Fall Test Paper
Complete the quiz in 75 minutes. Do the quiz as if you are taking the real test. You score will be compared with other students taking the same test to give you a ranking among your peers.
To get a human or AI tutor to help you, click Register
Sample Question 8:

Let \(M\) be the least common multiple of all the integers \(10\) through \(30,\) inclusive. Let \(N\) be the least common multiple of \(M,32,33,34,35,36,37,38,39,\) and \(40.\) What is the value of \(\frac{N}{M}?\)

\(\textbf{(A)}\ 1 \qquad\textbf{(B)}\ 2 \qquad\textbf{(C)}\ 37 \qquad\textbf{(D)}\ 74 \qquad\textbf{(E)}\ 2886\)




Answer Keys

Question 8: D


Solutions

Question 8
Step 1: We start by defining the Least Common Multiple (LCM). The LCM of a set of numbers is obtained by taking the greatest powers of the prime numbers in the prime factorization of all the numbers.

Step 2: Applying this rule, we determine that \(M = 2^4 \cdot 3^3 \cdot 5^2 \cdot 7 \cdot 11 \cdot 13 \cdot 17 \cdot 19 \cdot 23 \cdot 29.\), which is the least common multiple of all the integers from 10 to 30 inclusive.

Step 3: We then find the value of \(N\), which is the LCM of \(M\), 32, 33, 34, 35, 36, 37, 38, 39, and 40. Following the same rule, \(N = M \cdot 2 \cdot 37\). This is because there is an additional power of \(2\) and an additional power of \(37\) in the prime factorization of the set of numbers.

Step 4: Finally, we compute the value of \(\frac{N}{M}\) by substitute the found expression for \(N\) and \(M\). This becomes \(\frac{N}{M} = 2\cdot 37 = 74\).

So, the answer is \(\textbf{(D)}\ 74\).